Peds Test 3 Practice Questions

Pataasin ang iyong marka sa homework at exams ngayon gamit ang Quizwiz!

61. Which has the potential to alter a child's level of consciousness? Select all that apply. 1. Metabolic disorders. 2. Trauma. 3. Hypoxic episode. 4. Dehydration. 5. Endocrine disorders.

61. 1, 2, 3, 4, 5. 1. Many metabolic disorders are associated with hypoglycemia. The hypoglycemic child experiences a decreased level of consciousness as the brain does not have stores of glucose. 2. Trauma can lead to generalized brain swelling with resultant increased ICP. 3. Hypoxemia leads to a decreased level of consciousness as the brain is intolerant to the lack of oxygen. 4. Dehydration can lead to inadequate perfusion to the brain, which can result in a decreased level of consciousness. 5. Endocrine disorders often result in a decreased level of consciousness as they can lead to hypoglycemia, which is poorly tolerated by the brain.

Reye's syndome affects the: 1.) stomach and the intestine 2.) islet of Langerhans 3.) Liver and the brain 4.) heart and the blood vessels

LIver and brain

The nurse is collecting data on a child admitted to the burn unit with a partial-thickness burn. Which of the following is most accurate regarding this type of burn? a) There is no destruction of tissue. b) The child will have minimal pain. c) The child will likely have blisters. d) The nerve ending are destroyed.

c)The child will likely have blisters. Explanation: In a partial-thickness or second-degree burn, the epidermis and underlying dermis are both injured and devitalized or destroyed. Blistering usually occurs with an escape of body plasma, but regeneration of the skin occurs from the remaining viable epithelial cells in the dermis.

Which of the following interventions is the most effective in treating burn wound infections? a) Systemic intravenous antibiotics b) Proper hand washing c) Topical antibiotics applied to the wound site d) Systemic oral antibiotics

c)Topical antibiotics applied to the wound site Explanation: Topical burn creams are used because the local blood supply to the area of burn injury is destroyed with the burn, and systemic antibiotics thus are not delivered to the burn wound. Proper hand washing is a preventive treatment.

74. The parents of an infant with hydrocephalus ask about future activities in which their child can participate in school and as an adolescent. The nurse should tell the parents which of the following is appropriate? 1. A helmet should be worn during any activity that could lead to head injury. 2. Only non-contact sports should be pursued, such as swimming or tennis. 3. Because of the risk of shunt system infection, swimming is not a sports option. 4. The child should wear a MedicAlert bracelet; then there is no need to be concerned about the shunt.

1,2,3,5

Which child may need extra fluids to prevent dehydration? Select all that apply. 1. 7-day-old receiving phototherapy. 2. 6-month-old with newly diagnosed pyloric stenosis. 3. 2-year-old with pneumonia. 4. 2-year-old with full-thickness burns to the chest, back, and abdomen. 5. 13-year-old who has just started her menses.

1. 7-day-old receiving phototherapy. 2. 6-month-old with newly diagnosed pyloric stenosis. 3. 2-year-old with pneumonia. 4. 2-year-old with full-thickness burns to the chest, back, and abdomen.

4. A 4-month-old is brought to the emergency department with severe dehydration. The heart rate is 198, and her blood pressure is 68/38. The infant's anterior fontanel is sunken. The nurse notes that the infant does not cry when the intravenous line is inserted. The child's parents state that she has not "held anything down" in 18 hours. The nurse obtains a finger-stick blood sugar of 94. Which would the nurse expect to do immediately? 1. Administer a bolus of normal saline. 2. Administer a bolus of D10W. 3. Administer a bolus of normal saline with 5% dextrose added to the solution. 4. Offer the child an oral rehydrating solution such as Pedialyte.

1. Administer a bolus of normal saline

A 4-month-old is brought to the emergency department with severe dehydration. The heart rate is 198, and her blood pressure is 68/38. The infant's anterior fontanel is sunken. The nurse notes that the infant does not cry when the intravenous line is inserted. The child's parents state that she has not "held anything down" in 18 hours. The nurse obtains a finger-stick blood sugar of 94. Which would the nurse expect to do immediately? 1. Administer a bolus of normal saline. 2. Administer a bolus of D10W. 3. Administer a bolus of normal saline with 5% dextrose added to the solution. 4. Offer the child an oral rehydrating solution such as Pedialyte.

1. Administer a bolus of normal saline. (isotonic)

10. A child is being admitted with the diagnosis of meningitis. Select the procedure the nurse should do first: 1. Administration of intravenous antibiotics. 2. Administration of maintenance intravenous fluids. 3. Placement of a Foley catheter. 4. Send the spinal fluid and blood samples to the laboratory for cultures.

1. Administration of intravenous antibiotics should not be started until after all cultures have been obtained. 2. Administration of maintenance IV fluids can wait until after the cultures have been obtained. 3. Placement of a Foley catheter is not a priority procedure. 4. CORRECT Cultures of spinal fluid and blood should be obtained, followed by administration of intravenous antibiotics.

A 6-month-old is prescribed 2.5% hydrocortisone for topical treatment of eczema. The nurse instructs the parent not to use the cream for more than a week. What is the primary reason for this instruction? 1. Adverse effects, such as skin atrophy and fragility, can occur with long-term treatment. 2. If after a week there is no improvement, then a stronger dose is required. 3. The drug loses its efficacy after prolonged use. 4. If no improvement is seen after a week, an antibiotic should be prescribed.

1. Adverse effects, such as skin atrophy and fragility, can occur with long-term treatment.

33. A 2-month-old infant is brought to the emergency room after experiencing a seizure. The infant appears lethargic with very irregular respirations and periods of apnea. The parents report the baby is no longer interested in feeding and before the seizure, rolled off the couch. What additional testing should the nurse immediately prepare for? 1. Computed tomography (CT) scan of the head and dilation of the eyes. 2. Computed tomography (CT) scan of the head and electroencephalogram (EEG). 3. X-rays of the head. 4. X-rays of all long bones.

1. Computed tomography (CT) scan of the head and dilation of the eyes.

82. Which is/are the most common, nonlethal complication(s) occurring from meningitis? 1. Cranial nerve deficits. 2. Epilepsy. 3. Bleeding intracranially. 4. Cerebral palsy.

1. Cranial nerve deficits.

38. School-age children with cancer often have a body image disturbance related to hair loss, moon face, or debilitation. Which of the following interventions is most appropriate? 1. Encourage them to wear a wig similar to their own hairstyle. 2. Emphasize the benefits of the therapy they are receiving. 3. Have them play only with other children with cancer. 4. Use diversional techniques to avoid discussing changes in the body because of the chemotherapy.

1. Encourage them to wear a wig similar to their own hairstyle.

13. Which order would the nurse question for a child just admitted with the diagnosis of bacterial meningitis? 1. Maintain isolation precautions until 24 hours after receiving intravenous antibiotics. 2. Intravenous fluids at 11/2 times regular maintenance. 3. Neurological checks every hour. 4. Administer acetaminophen for temperatures higher than 38°C (100.4°F).

1. Isolation precautions must be maintained for at least the first 24 hours of intravenous antibiotic therapy. 2. CORRECT Intravenous fluids at 11/2 times regular maintenance could cause fluid overload and lead to increased ICP. 3. Neurological checks are usually made at least every hour. 4. Acetaminophen is usually administered when the child has a fever, as increased temperature can lead to increased ICP.

51. The parents of a 12-month-old with cerebral palsy (CP) ask the nurse if they should teach their child sign language because he has not begun to vocalize. The nurse bases the response on the knowledge that sign language: 1. May be a very beneficial way to help children with CP communicate. 2. May cause confusion and further delay vocalization. 3. Is difficult to learn for most children with CP. 4. Is beneficial to learn, but it would be best to wait until the child is older

1. May be a very beneficial way to help children with CP communicate.

25. Which activity should an adolescent just diagnosed with epilepsy avoid? 1. Swimming, even with a friend. 2. Being in a car at night. 3. Participating in any strenuous activities. 4. Returning to school right away.

1. Swimming does not need to be avoided as long as there is someone else present to call for help in the event of an emergency. 2. CORRECT The rhythmic reflection of other car lights can trigger a seizure in some children. 3. There is no reason to avoid strenuous activity. 4. It is important for adolescents to be with their peers in order to reach developmental milestones.

40. The nurse is caring for a child being treated for ALL. Laboratory results indicate that the child has a white blood cell count of 5000/mm3 with 5% polys and 3% bands. Which of the following analyses is most appropriate? 1. The absolute neutrophil count is 400/mm3, and the child is neutropenic. 2. The absolute neutrophil count is 800/mm3, and the child is neutropenic. 3. The absolute neutrophil count is 4000/mm3, and the child is not neutropenic 4. The absolute neutrophil count is 5800/mm3, and the child is not neutropenic.

1. The absolute neutrophil count is 400/mm3, and the child is neutropenic.

18. A child diagnosed with meningitis is having a generalized tonic-clonic seizure. Which should the nurse do first? 1. Administer blow-by oxygen and call for additional help. 2. Reassure the parents that seizures are common in children with meningitis. 3. Call a code and ask the parents to leave the room. 4. Assess the child's temperature and blood pressure.

1. The child experiencing a seizure usually requires more oxygen as the seizure increases the body's metabolic rate and demand for oxygen. The seizure may also affect the child's airway, causing the child to be hypoxic. It is always appropriate to give the child blow-by oxygen immediately. The nurse should remain with the child and call for additional help.

12. Which of the following measures should the nurse implement to help with the nausea and vomiting from chemotherapy? Select all that apply. 1. Give an antiemetic 30 minutes prior to the start of therapy. 2. Continue the antiemetic as ordered until 24 hours after the chemotherapy is complete. 3. Remove food that has a lot of odor. 4. Keep the child on a nothing-by-mouth status. 5. Wait until the nausea begins to start the antiemetic.

1. The first dose should be given 30 minutes prior to the start of the therapy. 2. Antiemetic should be administered around the clock until 24 hours after the chemotherapy is completed. 3. It is also helpful to remove foods with odor so the smell of the food does not make the child nauseated.

17. To treat a common manifestation of Reye syndrome, which medication would the nurse expect to have readily available? 1. Lasix. 2. Insulin. 3. Glucose. 4. Morphine.

17. 1. A common manifestation is increased ICP, which is treated with an osmotic diuretic. Lasix is a loop diuretic. 2. A common manifestation is hypoglycemia. Insulin does not treat hypoglycemia, but decreases the blood sugar instead. 3. CORRECT A common manifestation is hypoglycemia, which is treated with the administration of intravenous glucose. 4. Morphine is a narcotic used for pain relief. It should be used with caution as it can lead to respiratory depression.

6. Which of the following would be included in the plan of care for a hospitalized newborn following surgical repair of a myelomeningocele. Select all that apply. 1. Skull x-rays. 2. Daily head circumference measurements. 3. MRI scan. 4. Vital signs every 6 hours. 5. Holding to breastfeed.

2, 3. 1. Diagnostic tests include MRI scan, CT scan, ultrasound, and myelography. These tests give much more needed information than do skull x-rays. 2. Daily head circumference measurements are done to assess for hydrocephalus. 3. Diagnostic tests include MRI scan, CT scan, ultrasound, and myelography. 4. Vital signs should be taken at least every 4 hours. 5. Infants with repaired myelomeningocele are kept prone to prevent pressure on the surgical site.

35. The nurse is caring for a newborn with a cleft lip and palate. The mother states, "I will not be able to breastfeed my baby." Select the nurse's best response. 1. "It sounds like you are feeling discouraged. Would you like to talk about it?" 2. "Sometimes breastfeeding is still an option for babies with a cleft lip and palate. Would you like more information?" 3. "Although breastfeeding is not an option, you have the option of pumping your milk and then feeding it to your baby with a special nipple." 4. "We usually discourage breastfeeding babies with cleft lip and palate as it puts them at an increased risk for aspiration."

2. "Sometimes breastfeeding is still an option for babies with a cleft lip and palate. Would you like more information?"

45. Which child is at increased risk for cerebral palsy (CP)? 1. Infant born at 34 weeks with an Apgar score of 6 at 5 minutes. 2. 17-day-old infant with group B streptococcus meningitis. 3. 24-month-old child who has experienced a febrile seizure. 4. 5-year-old with a closed-head injury after falling off a bike.

2. 17-day-old infant with group B streptococcus meningitis.

44. A 3-year-old child with CP is admitted for dehydration following an episode of diarrhea. The nurse's assessment follows: awake, pale, thin child lying in bed, multiple contractures, drooling, coughing spells noted when parent feeds. T 97.8°F (36.5°C), P 75, R 25, weight 7.2 kg, no diarrheal stool for 48 hours. Which nursing diagnosis is most important? 1. Potential for skin breakdown: lying in one position. 2. Alteration in nutrition: less than body requirements. 3. Potential for impaired social support: mother sole caretaker. 4. Alteration in elimination: diarrhea.

2. Alteration in nutrition: less than body requirements.

24. Which needs to be present to diagnose hemolytic uremic syndrome (HUS)? 1. Increased red blood cells with a low reticulocyte count, increased platelet count, and renal failure. 2. Decreased red blood cells with a high reticulocyte count, decreased platelet count, and renal failure. 3. Increased red blood cells with a high reticulocyte count, increased platelet count, and renal failure. 4. Decreased red blood cells with a low reticulocyte count, decreased platelet count, and renal failure.

2. Decreased red blood cells with a high reticulocyte count, decreased platelet count, and renal failure.

84. During the nurse's assessment, a child begins to have a generalized tonic-clonic seizure. The drug of choice and method of administration the nurse expects the health-care provider (HCP) to order are which of the following? 1. Lorazepam and diazepam, combined in an intravenous solution of D5W. 2. Lorazepam given intravenously or diazepam given directly into a vein. 3. Phenobarbital administered in an intravenous solution of D5W, 0.45 normal saline. 4. Phenytoin in a dextrose solution given intravenously over 1 hour.

2. Lorazepam given intravenously or diazepam given directly into a vein.

33. The nurse is caring for a 10-year-old with leukemia who is receiving chemotherapy. The child is on neutropenic precautions. Friends of the child come to the desk and ask for a vase for flowers. Which of the following is the best response? 1. "I will get you a special vase that we use on this unit." 2. "The flowers from your garden are beautiful but should not be placed in the room at this time." 3. "As soon as I can wash a vase, I will put the flowers in it and bring it to the room." 4. "Get rid of the flowers immediately. You could harm the child."

2. NO DAMN FLOWERS

Lindane (G-Well) shampoo is used only once because it can cause: 1. Hypertension. 2. Seizures. 3. Elevated liver functions. 4. Alopecia.

2. Seizures Lindane (Kwell) with topical use is associated with seizures after absorption.

9. The nurse is aware that cloudy cerebrospinal fluid (CSF) most likely indicates: 1. Viral meningitis. 2. Bacterial meningitis. 3. No infection, as CSF is usually cloudy. 4. Sepsis.

2. The CSF of bacterial meningitis is usually cloudy

38. A parent of a newborn diagnosed with myelomeningocele asks what is a common long-term complication? The nurse's best response is: 1. Learning disabilities. 2. Urinary tract infections. 3. Hydrocephalus. 4. Decubitus ulcers and skin breakdown.

2. Urinary tract complications

In infants with hydrocephalus, the early signs of ventricular shunt malfunction are: 1. a high-pitched cry, colic, and pupillary changes. 2. a tense fontanelle, vomiting, and irritability. 3. anorexia, and changes in pulse and respiration. 4. headaches, lethargy, and a loss of appetite.

2. a tense fontanelle, vomiting, and irritability.

35. An infant is born with a sac protruding through the spine, containing cerebrospinal fluid (CSF), a portion of the meninges, and nerve roots. This condition is referred to as: 1. Meningocele. 2. Myelomeningocele. 3. Spina bifida occulta. 4. Anencephaly.

2. myelomeningocele

After surgery for pyloric stenosis, the nurse could anticipate that the infant will: 1.) have nasogastric suction for 24 hours 2.) be fed clear liquids within 6 hours 3.) remain NPO for 24 to 48 hours 4.) be fed formula within 4 hours

2.) be fed clear liquids within 6 hours

20. Which would the nurse most likely find in the history of a child with hemolytic uremic syndrome (HUS)? 1. Frequent UTIs and possible vesicoureteral reflux (VUR). 2. Vomiting and diarrhea before admission. 3. Bee sting and localized edema of the site for 3 days. 4. Previously healthy and no signs of illness.

20. 1. Frequent UTIs and VUR do not lead to HUS. 2. CORRECT HUS is often preceded by diarrhea that may be caused by E. coli present in undercooked meat. 3. Insect stings are not associated with HUS. 4. HUS is usually preceded by diarrhea.

24. A child with a repaired myelomeningocele is in the clinic for a regular examination. The child has frequent constipation and has been crying at night because of pain in the legs. After an MRI, the diagnosis of a tethered cord is made. Which should the nurse tell the parent? 1. Tethered cord is a post-surgical complication. 2. Tethered cord occurs during times of slow growth. 3. Release of the tethered cord will be necessary only once. 4. Offering laxatives and acetaminophen daily will help control these problems.

24. 1. CORRECT Tethered cord is caused by scar tissue formation from the surgical repair of the myelomeningocele and may affect bowel, bladder, or lower extremity functioning. 2. Tethered cord occurs during growth spurts. 3. Often the release of the tether will again become necessary. 4. Laxatives and acetaminophen are temporary remedies, and they treat only the symptoms.

The pathologic disturbance of pyloric stenosis results from: 1.) edema of the pyloric muscle 2.) ischemia of the pyloric muscle 3.) hypertrophy of the pyloric muscle 4.) neoplastic obstruction

3) hypertrophy of the pyloric muscle

49. The nurse prepares to administer baclofen to a child with cerebral palsy (CP) who just had her hamstrings surgically released. The child's parents ask what the medication is for. Select the nurse's best response. 1. "It is a medication that will help decrease the pain from her surgery." 2. "It is a medication that will prevent her from having seizures." 3. "It is a medication that will help control her spasms." 4. "It is a medication that will help with bladder control."

3. "It is a medication that will help control her spasms."

45. The parent of an infant with CP asks the nurse if the infant will be mentally retarded. Which is the nurse's best response? 1. "Children with CP have some amount of mental retardation." 2. "Approximately 20% of children with CP have normal intelligence." 3. "Many children with CP have normal intelligence." 4. "Mental retardation is expected if motor and sensory deficits are severe."

3. "Many children with CP have normal intelligence."

45. The nurse is caring for an 8-week-old infant being evaluated for pyloric stenosis. Which statement by the parent would be typical for a child with this diagnosis? 1. "The baby is a very fussy eater and just does not want to eat." 2. "The baby tends to have a very forceful vomiting episode about 30 minutes after most feedings." 3. "The baby is always hungry after vomiting so I refeed." 4. "The baby is happy in spite of getting really upset after spitting up."

3. "The baby is always hungry after vomiting so I refeed."

48. The nurse is caring for a 5-month-old infant with a diagnosis of intussusception. The infant has periods of irritability during which the knees are brought to chest and the infant cries, alternating with periods of lethargy. Vital signs are stable and within age-appropriate limits. The physician elects to give an enema. The parents ask the purpose of the enema. Select the nurse's most appropriate response. 1. "The enema will confirm the diagnosis. If the test result is positive, your child will need to have surgery to correct the intussusception." 2. "The enema will confirm the diagnosis. Although very unlikely, the enema may also help fix the intussusception so that your child will not immediately need surgery." 3. "The enema will help confirm the diagnosis and has a good chance of fixing the intussusception." 4. "The enema will help confirm the diagnosis and may temporarily fix the intussusception. If the bowel returns to normal, there is a strong likelihood that the intussusception will recur."

3. "The enema will help confirm the diagnosis and has a good chance of fixing the intussusception."

47. The parents of a child with meningitis and multiple seizures ask if the child will likely develop cerebral palsy (CP). Select the nurse's best response. 1. "When your child is stable, she'll undergo computed tomography (CT) and magnetic resolution imaging (MRI). The physicians will be able to let you know if she has CP." 2. "Most children do not develop CP at this late age." 3. "Your child will be closely monitored after discharge, and a developmental specialist will be able to make the diagnosis." 4. "Most children who have had complications following meningitis develop some amount of CP."

3. "Your child will be closely monitored after discharge, and a developmental specialist will be able to make the diagnosis."

5. The nurse is caring for a 2-year-old child who was admitted to the pediatric unit for moderate dehydration due to vomiting and diarrhea. The child is restless, with periods of irritability. The child is afebrile with a heart rate of 148 and a blood pressure of 90/42. Baseline laboratory tests reveal the following: Na 152, Cl 119, and glucose 115. The parents state that the child has not urinated in 12 hours. After establishing a saline lock, the nurse reviews the physician's orders. Which order should the nurse question? 1. Administer a saline bolus of 10 mL/kg, which may be repeated if the child does not urinate. 2. Recheck serum electrolytes in 12 hours. 3. After the saline bolus, begin maintenance fluids of D5 1/4 NS with 10 mEq KCl/L. 4. Give clear liquid diet as tolerated.

3. After the saline bolus, begin maintenance fluids of D5 1/4 NS with 10 mEq Kcl/L

22. Which laboratory results besides hematuria are most consistent with hemolytic uremic syndrome (HUS)? 1. Massive proteinuria, elevated blood urea nitrogen, and creatinine. 2. Mild proteinuria, decreased blood urea nitrogen, and creatinine. 3. Mild proteinuria, increased blood urea nitrogen, and creatinine. 4. Massive proteinuria, decreased blood urea nitrogen, and creatinine.

3. Hematuria, mild proteinuria, increased BUN, and creatinine are all present in HUS.

15. The nurse is caring for a 3-month-old being evaluated for possible Hirschsprung disease. His parents call the nurse and show her his diaper containing a large amount of mucus and bloody diarrhea. The nurse notes that the infant is irritable and his abdomen appears very distended. Which should be the nurse's next action? 1. Reassure the parents that this is an expected finding and not uncommon. 2. Call a code for a potential cardiac arrest, and stay with the infant. 3. Immediately obtain all vital signs with a quick head-to-toe assessment. 4. Obtain a stool sample for occult blood.

3. Immediately obtain all vital signs with a quick head-to-toe assessment.

25. Which should be included in the plan of care for a newborn with a myelomeningocele who will have a surgical repair tomorrow? 1. Offer formula every 3 hours. 2. Turn the infant back to front every 2 hours. 3. Place a wet dressing on the sac. 4. Provide pain medication every 4 hours.

3. Place a wet dressing on the sac.

72. Which is immediate post-operative care for shunt placement in an infant diagnosed with hydrocephalus? 1. Wet-to-dry dressing changes at both the shunt insertion site and the abdominal incision site. 2. Measure the child's head at least once a day. 3. Position the infant's head off the shunt site for the first 2 post-operative days. 4. Complete vital signs and neurological checks every 4 hours.

3. Position the infant's head off the shunt site for the first 2 post-operative days.

75. While assisting with a lumbar puncture procedure on an infant or small child, the nurse should do which of the following? 1. Have the patient in a clean diaper to avoid contamination of the site. 2. Monitor the patient's cardiorespiratory status at all times. 3. Position the patient in the prone position with the head to the left. 4. Start an intravenous line to facilitate use of conscious sedation.

3. Position the infant's head off the shunt site for the first 2 post-operative days.

2. The nurse is caring for a 4-year-old who weighs 15 kg. At the end of a 10-hour period, the nurse notes the urine output to be 150 mL. What action does the nurse take? 1. Notifies the physician because this urine output is too low. 2. Encourages the child to increase oral intake to increase urine output. 3. Records the child's urine output in the chart. 4. Administers isotonic fluid intravenously to help with rehydration.

3. Records the child's urine output in the chart. Because the output is appropriate for the given time period 15 ml an hour in a 10 hour period is 150 mL

32. A 5-year-old is admitted to the hospital with complaints of leg pain and fever. On physical examination, the child is pale and has bruising over various areas of the body. The physician suspects that the child has ALL. The nurse informs the parent that the diagnosis will be confirmed by which of the following? 1. Lumbar puncture. 2. White blood cell count. 3. Bone marrow aspirate. 4. Bone scan.

3. The diagnostic test that confirms leukemia is microscopic examination of the bone marrow aspirate.

99. Which applies to encephalitis? Select all that apply. 1. Usually caused by a bacterial infection. 2. A chronic disease. 3. Most commonly seen after a varicella infection in the newborn population. 4. Newborns diagnosed with encephalitis often have extensive neurological problems. 5. Can be seen with meningitis.

4,5

34. The nurse is discharging a child who has just received chemotherapy for neuroblastoma. Which of the following statements made by the child's parent indicates a need for additional teaching? 1. "I will inspect the skin often for any lesions." 2. "I will do mouth care daily and monitor for any mouth sores." 3. "I will wash my hands before caring for my child." 4. "I will take a rectal temperature daily and report a temperature greater than 101°F (38.3°C) immediately to the physician."

4. "I will take a rectal temperature daily and report a temperature greater than 101°F (38.3°C) immediately to the physician."

34. The parents of a newborn diagnosed with a cleft lip and palate ask the nurse when their child's lip and palate will most likely be repaired. Select the nurse's best response. 1. "The palate and the lip are usually repaired in the first few weeks of life so that the baby can grow and gain weight." 2. "The palate and the lip are usually not repaired until the baby is approximately 6 months old so that the mouth has had enough time to grow." 3. "The lip is repaired in the first few months of life, but the palate is not usually repaired until the child is 3 years old." 4. "The lip is repaired in the first few weeks of life, but the palate is not usually repaired until the child is 18 months old."

4. "The lip is repaired in the first few weeks of life, but the palate is not usually repaired until the child is 18 months old."

37. When teaching parents about osteosarcoma, the nurse knows instruction has been successful when a parent says that this type of cancer is common in which age group? 1. Infancy. 2. Toddlers. 3. School-aged children. 4. Adolescents.

4. Adolescents

36. The nurse evaluates teaching of parents of a child newly diagnosed with cerebral palsy (CP) as successful when the parents state that CP is which of the following? 1. Inability to speak and uncontrolled drooling. 2. Involuntary movements of lower extremities only. 3. Involuntary movements of upper extremities only. 4. An increase in muscle tone and deep tendon reflexes.

4. An increase in muscle tone and deep tendon reflexes.

27. Brain damage in a child who sustained a closed-head injury can be caused by which factor? 1. Increased perfusion to the brain and increased metabolic needs of the brain. 2. Decreased perfusion to the brain and decreased metabolic needs of the brain. 3. Increased perfusion to the brain and decreased metabolic needs of the brain. 4. Decreased perfusion of the brain and increased metabolic needs of the brain.

4. Decreased perfusion of the brain and increased metabolic needs of the brain.

17. Which should the nurse include in the plan of care to decrease symptoms of gastroesophageal reflux (GER) in a 2-month-old? Select all that apply. 1. Place the infant in an infant seat immediately after feedings. 2. Place the infant in the prone position immediately after feeding to decrease the risk of aspiration. 3. Encourage the parents not to worry because most infants outgrow GER within the first year of life. 4. Encourage the parents to hold the infant in an upright position for 30 minutes following a feeding. 5. Suggest that the parents burp the infant after every 1-2 ounces consumed.

4. Encourage the parents to hold the infant in an upright position for 30 minutes following a feeding. 5. Suggest that the parents burp the infant after every 1-2 ounces consumed.

35. The nurse is developing a plan of care for a child recently diagnosed with cerebral palsy (CP). Which should be the nurse's priority goal? 1. Ensure the ingestion of sufficient calories for growth. 2. Decrease intracranial pressure. 3. Teach appropriate parenting strategies for a special-needs child. 4. Ensure that the child reaches full potential.

4. Ensure that the child reaches full potential.

47. Which is the primary goal for a newborn with a cleft of the soft palate? 1. Prevent ear infections. 2. Help the mother bond with the baby. 3. Repair the cleft palate. 4. Establish feeding and sucking.

4. Establish feeding and sucking.

33. Which should the nurse prepare the parents of an infant for following surgical repair and closure of a myelomeningocele shortly after birth? The infant will: 1. Not need any long-term management and should be considered cured. 2. Not be at risk for urinary tract infections or movement problems. 3. Have continual drainage of cerebrospinal fluid, needing frequent dressing changes. 4. Need lifelong management of urinary, orthopedic, and neurological problems.

4. Need lifelong management of urinary, orthopedic, and neurological problems.

31. A 15-year-old with spina bifida is seen in the clinic for a well-child checkup. The teen uses leg braces and crutches to ambulate. Which nursing diagnosis takes priority? 1. Potential for infection. 2. Alteration in mobility. 3. Alteration in elimination. 4. Potential body image disturbance.

4. Potential body image disturbance.

49. The nurse will soon receive a 4-month-old who has been diagnosed with intussusception. The infant is described as very lethargic with the following vital signs: T 101.8°F (38.7°C), HR 181, BP 68/38. The reporting nurse states the infant's abdomen is very rigid. Which is the most appropriate action for the receiving nurse? 1. Prepare to accompany the infant to a computed tomography scan to confirm the diagnosis. 2. Prepare to accompany the infant to the radiology department for a reducing enema. 3. Prepare to start a second intravenous line to administer fluids and antibiotics. 4. Prepare to get the infant ready for immediate surgical correction.

4. Prepare to get the infant ready for immediate surgical correction.

1. An adolescent presents with sudden-onset unilateral facial weakness with drooping of one side of the mouth. The teen is unable to close the eye on the affected side, but has no other symptoms and otherwise feels well. The nurse could summarize the condition by which of the following? 1. The prognosis is poor. 2. This may be a stroke. 3. It is a fifth CN palsy. 4. This is paralysis of the facial nerve.

4. This is paralysis of the facial nerve.

71. A newborn with suspected hydrocephalus is transferred to the intensive care unit for further evaluation and treatment. The infant's nurse knows which of the following? 1. To use sedation as needed to keep the baby from crying or being fussy. 2. To keep the crib in a flat and neutral position. 3. To expect the infant to sleep more than an infant without hydrocephalus. 4. To not use any scalp veins for intravenous infusions.

4. To not use any scalp veins for intravenous infusions.

10. After airway, breathing, and circulation have been assessed and stabilized, which intervention should the nurse implement for a child diagnosed with encephalitis? 1. Assist with a lumbar puncture, and give reassurance. 2. Obtain a throat culture, then begin antibiotics. 3. Perform initial and serial neurological assessments. 4. Administer antibiotics and antipyretics.

Airway, breathing, and circulation are part of the primary patient assessment. Neurological assessment is the next assessment to perform. 38 PEDIATRIC SUCCESS 2. The child has been diagnosed with encephalitis. Unless there is a concern about the child's having strep throat, a throat culture would not be obtained. 3. CORRECT Initial and serial neurological assessments would be a priority nursing intervention in a child with a neurological problem. It is to monitor for changes in neurological status. 4. Encephalitis is usually caused by a virus therefore, antibiotics are not ordered. Antipyretics may be used to help control fevers.

An infant is experiencing dyspnea related to patent ductus arteriosus (PDA), The nurse understands dyspnea occurs because blood is

An infant is experiencing dyspnea related to patent ductus arteriosus (PDA), The nurse understands dyspnea occurs because blood is Circulated through the lungs again, causing pulmonary circulatory congestion.

The nurse explains that the treatment of choice for a child with intussusception is

Barium Enema This would be used to diagnose it and could possibly cure it.

52. The parents of a child with cerebral palsy (CP) are learning how to feed their child and avoid aspiration. The nurse would question which of the following when reviewing the teaching plan? 1. Place the food on the tip of the tongue. 2. Place the child in an upright position during feedings. 3. Feed the child soft and blended foods. 4. Feed the child slowly.

1. Place the food on the tip of the tongue.

42. A 6-month-old infant was just diagnosed with craniosynostosis. The infant's father asks the nurse for more information about reconstructive surgery. Select the nurse's best response. 1. "The surgery is done for cosmetic reasons and is without many complications." 2. "The surgery is important to allow the brain to grow properly. Although most children do well, serious complications can occur, so your child will be closely observed in the intensive care unit." 3. "The surgery is important to allow the brain to grow properly. Most surgeons wait until the child is 3 years old to minimize potential complications." 4. "The surgery is mainly done for cosmetic reasons, and most surgeons wait until the child is 3 years old as the head has finished growing at that time."

2. "The surgery is important to allow the brain to grow properly. Although most children do well, serious complications can occur, so your child will be closely observed in the intensive care unit."

6. An appropriate nursing action related to the administration of digoxin (Lanoxin) to an infant would be: a. counting the apical rate for 30 seconds before administering the medication. b. withholding a dose if the apical heart rate is less than 100 beats/min. c. repeating a dose if the child vomits within 30 minutes of the previous dose. d. checking respiratory rate and blood pressure before each dose.

ANS: B As a rule, if the pulse rate of an infant is below 100 beats/min, the medication is withheld and the physician is notified.

3. A child had a urinary tract infection (UTI) 3 months ago and was treated with an oral antibiotic. A follow-up urinalysis revealed normal results. The child has had no other problems until this visit when the child was diagnosed with another UTI. Which is the most appropriate plan? 1. Urinalysis, urine culture, and VCUG. 2. Evaluate for renal failure. 3. Admit to the pediatric unit. 4. Discharge home on an antibiotic.

.1. Urinalysis and urine culture are routinely used to diagnose UTIs. VCUG is used to determine the extent of urinary tract involvement when a child has a second UTI within 1 year.

Which would the nurse instruct a parent to apply to treat a pediculosis infestation? 1. Lindane (Kwell) to the scalp, leaving it in place for 4 minutes, and then adding water. 2. Chlorhexidine (Hibiclens) to the scalp with sterile gloves. 3. Terbinafine (Lamisil) as a thin layer to the scalp twice a day. 4. Collagenase (Santyl) to the scalp with cotton applicator.

1. Lindane (Kwell) to the scalp, leaving it in place for 4 minutes, and then adding water.

Which is assessed to diagnose pediculosis capitis? Select all that apply. 1. Crawling insects. 2. White flakes in the hair. 3. Nits attached close to scalp. 4. Inflammatory papules. 5. Dark brown hair.

1. crawling insects 3. nits attached close to scalp 4. inflammatory papules

A parent asks the nurse to tell her what cerebral palsy is. The best response of the nurse would be that it is a: 1.) motor disability caused by a nonprogressive disturbance in brain development 2.) disorder of the brain that results in mental retardation 3.) complication of the birth process that causes brain damage 4.) brain disorder that involves seizures

1.) motor disability caused by a nonprogressive disturbance in brain development

29. Which should the nurse tell the parent of an infant with spina bifida? 1. "Bone growth will be more than that of babies who are not sick because your baby will be less active." 2. "Physical and occupational therapy will be helpful to stimulate the senses and improve cognitive skills." 3. "Nutritional needs for your infant will be calculated based on activity level." 4. "Fine motor skills will be delayed because of the disability."

2. "Physical and occupational therapy will be helpful to stimulate the senses and improve cognitive skills."

28. Over the last week, an infant with a repaired myelomeningocele has had a highpitched cry and been irritable. Length, weight, and head circumference have been at the 50th percentile. Today length is at the 50th percentile, weight is at the 70th percentile, and head circumference is at the 90th percentile. The nurse should do which of the following? 1. Tell the parent this is normal for an infant with a repaired myelomeningocele. 2. Tell the parent this might mean the baby has increased intracranial pressure. 3. Suspect the baby's intracranial pressure is low because of a leak. 4. Refer the baby to the neurologist for follow-up care.

2. Tell the parent this might mean the baby has increased intracranial pressure.

70. Which are early signs and symptoms of hydrocephalus in infants? 1. Confusion, headache, diplopia. 2. Rapid head growth, poor feeding, confusion. 3. Papilledema, irritability, headache. 4. Full fontanels, poor feeding, rapid head growth.

4. Full fontanels, poor feeding, rapid head growth.

A 3-month-old infant has a hypercyanotic spell. The nurse's first action should be which of the following? a. Assess for neurologic defects. b. Prepare family for imminent death. c. Begin cardiopulmonary resuscitation. d. Place child in the knee-chest position.

ANS :D

The most common type of burn in the toddler age-group is: a.electric burn from electrical outlets. b.flame burn from playing with matches. c.hot object burn from cigarettes or irons. d.scald burn from high-temperature tap water.

ANS: D

The nurse is caring for a school-age girl who has had a cardiac catheterization. The child tells the nurse that her bandage is "too wet." The nurse finds the bandage and bed soaked with blood. The most appropriate initial nursing action is which of the following? a. Notify the physician. b. Place child in Trendelenburg position. c. Apply a new bandage with more pressure. d. Apply direct pressure above catheterization site.

ANS: D

The appropriate technique for the application of a topical treatment for a child with eczema is to

Apply prescribed ointments after a bath.

An infant with congestive heart failure is receiving digoxin (Lanoxin). The nurse recognizes a sign of digoxin toxicity, which is: a. restlessness. b. decreased respiratory rate. c. increased urinary output. d. vomiting.

D. Vomiting

Which assessment of an 18-month-old with burns on his feet would cause suspicion of child abuse? 1. Splash marks on his right lower leg. 2. Burns noted on right arm. 3. Symmetrical burns on both feet. 4. Burns mainly noted on right foot.

Physical abuse has certain characteristics. Symmetrical burns on both feet indicate abuse.

The nurse is caring for a 2-year-old child who was admitted to the pediatric unit for moderate dehydration due to vomiting and diarrhea. The child is restless, with periods of irritability. The child is afebrile with a heart rate of 148 and a blood pressure of 90/42. Baseline laboratory tests reveal the following: Na 152, Cl 119, and glucose 115. The parents state that the child has not urinated in 12 hours. After establishing a saline lock, the nurse reviews the physician's orders. Which order should the nurse question? 1. Administer a saline bolus of 10 mL/kg, which may be repeated if the child does not urinate. 2. Recheck serum electrolytes in 12 hours. 3. After the saline bolus, begin maintenance fluids of D5 1/4 NS with 10 mEq KCl/L. 4. Give clear liquid diet as tolerated.

Potassium is contraindicated because the child has not yet urinated. Potassium is not added to the maintenance fluid until kidney function has been verified.

The most recent blood count for a child who received chemotherapy last week shows neutropenia, The priority nursing diagnosis for this child is

Risk for Infection

When asked about correcting the hypospadias of a newborn, the nurse would explain that with this condition

Surgical repair of the hypospadias is done before 18 months of age. Hold off on circumcision until this is done

The nurse stresses to the mother of a child with impetigo that

The nurse stresses to the mother of a child with impetigo that Honey colored crusts on the lesions should be removed with a warm wash cloth

The nurse is providing teaching on ways to maintaining skin integrity and preventing infection for the parents of a boy with atopic dermatitis. Which of the following responses indicates a need for further teaching? a) "We should avoid using petroleum jelly." b) "We should avoid tight clothing and heat." c) "We need to develop ways to prevent him from scratching." d) "We should keep his fingernails short and clean."

a)"We should avoid using petroleum jelly." Explanation: It is important to apply moisture multiply times through the day. Petroleum jelly is a recommended moisturizer that is inexpensive and readily available. The other statements are correct.

The nurse is caring for a 6-year-old patient brought into the emergency department for burns from a house fire. The nurse notes burn areas surrounding the patient's nose and mouth upon initial assessment. Which of the following priority complications should the nurse be alerted to? a) Airway obstruction related to upper respiratory swelling b) Nutritional requirements increased c) Presence of an ileus d) One third area of fluid leakage resulting in hypovolemic shock

a)Airway obstruction related to upper respiratory swelling Explanation: Airway obstruction related to swelling is a priority complication to be alert for when signs of inhalation injury such as burns on the mouth and nose are present. Presence of an ileus, increased nutritional requirements, and hypovolemic shock are all complications of burns; however, airway obstruction is the priority.

In working with infants diagnosed with atopic dermatitis, the nurse anticipates that when these children are older they will likely have a tendency to have which of the following disorders? a) Asthma b) Rheumatoid arthritis c) Hemophilia d) Otitis media

a)Asthma Explanation: Infants who have eczema tend to have allergic rhinitis or asthma later in life.

The process of removing necrotic tissue in the treatment of burns is known as which of the following? a) Débridement b) Autograft c) Allograft d) Hydrotherapy

a)Débridement Explanation: Débridement (removal of necrotic tissue), usually preceded by hydrotherapy (use of water in treatment), is performed in the treatment of burns. Débridement is extremely painful, and the child must have an analgesic administered before the therapy.

The nurse is caring for a 1-year-old patient in a pediatric clinic. The patient was brought to the clinic with symptoms of dry, itchy red patches of skin on the arms and legs. A diagnosis of atopic dermatitis (eczema) is made. Which of the following is a key element in the treatment regimen for this diagnosis? a) Frequently rehydrating the skin b) Applying topical antibiotics routinely c) Teaching the child not to scratch the "itchy" skin d) Daily oral cortisone

a)Frequently rehydrating the skin Explanation: Frequently rehydrating the skin is a key element of the treatment regimen. To maintain healthy skin in the child with AD, hydration practices should be implemented to replace moisture in the stratum corneum and prevent transdermal water losses. Scratching the itchy skin is a reflex that is very difficult to stop; preventing the itch is more effective. Topical antibiotics and oral cortisone are not treatments for atopic dermatitis.

The nurse is discussing dietary intake with the parents of a 4-year-old child who has been diagnosed with atopic dermatitis. Later nurse notes the menu selection made by the parents for the child. Which selection indicates the need for further instruction? a) Peanut butter and jelly sandwich b) Carrot and celery sticks c) Tomato soup d) Chicken nuggets

a)Peanut butter and jelly sandwich Explanation: Atopic dermatitis is commonly associated allergies to food. Common culprits may include peanuts, eggs, orange juice, and wheat-containing products.

In caring for a 3 ½-year-old child admitted after being severely burned, the nurse collects the following data. Which of the following would be most important for the nurse to report immediately? a) The child's respiratory rate is 32 breaths a minute. b) The child's pain level is a 7 on the pain scale. c) The child's hourly urinary output is 150 cc. d) The child's temperature is 38.4°C.

a)The child's respiratory rate is 32 breaths a minute. Explanation: An increase in the respiratory rate after a severe burn may be an indication of possible serious respiratory complications and should be reported immediately in case an endotracheal tube needs to be inserted. A temperature of 38.4°C, hourly urine output of 150 cc, and pain rating of 7 need to be documented and reported but are not as urgent as reporting respiratory concerns.

A child is brought to the ER with burns on face and chest, The nurse's first priority is

assess Respiratory Status

A topical corticosteroid is prescribed for a child with contact dermatitis. Which statement by the mother would indicate the teaching was successful? a) "I should use the highest-potency steroid cream I can find." b) "I should not cover the area with plastic wrap after applying the cream." c) "I should apply the medicine at bedtime and rinse it off in the morning." d) "I need to shake the preparation before using it."

b)"I should not cover the area with plastic wrap after applying the cream." Explanation: An occlusive dressing such as plastic wrap over the area should not be used with topical corticosteroids. High-potency preparations should not be used. There is no need to shake topical corticosteroids. Benzoyl peroxide requires shaking before use. Applying the medication at night and rinsing off in the morning is used for coal tar preparations.

An 8-month-old has been diagnosed with infantile eczema. At a follow-up appointment, the child's caregiver seems exhausted and angry. He explains that he has done all of the child's care because his wife is repulsed by the child's raw and uncomfortable appearance. Which of the following responses would be appropriate for the nurse to say to this caregiver? a) "That's not fair to you; she should get some counseling to learn how to cope with illness better." b) "That's not an uncommon reaction, although it's hard on you and on your child." c) "He will be better soon and your family can get back to normal." d) "I understand her feelings. It is hard to see a child in pain sometimes."

b)"That's not an uncommon reaction, although it's hard on you and on your child." Explanation: The family caregivers of the child with eczema are often frustrated and exhausted. Family caregivers may feel apprehensive or repulsed by this unsightly child. Support them in expressing their feelings and help them view this as a distressing but temporary skin condition. Although the caregiver can be assured that most cases of eczema clear up by the age of 2, this does little to relieve the present situation.

A 16-year-old male who has diagnosed with tinea pedis questions the nurse about how he may have contracted the condition. What information may be provided to the boy by the nurse? a) "This condition is common in individuals with lowered immunity." b) "You may have gotten the condition from a community shower or gym area." c) "It is unlikely you will be able to determine the cause of the infection." d) "You likely had an infection in another area of your body and it has spread"

b)"You may have gotten the condition from a community shower or gym area." Explanation: Tinea pedis is commonly known as athlete's foot. It is a fungal infection. The fungi are able to readily grow in warm, moist conditions such as shower areas.

The nurse is conducting a physical examination of a child with severe burns. Which of the following internal physiologic manifestations would the nurse expect to occur first? a) Increased protein catabolism b) Decrease in cardiac output c) Hypermetabolic response with increased cardiac output d) Insulin resistance

b)Decrease in cardiac output Explanation: Initially, the severely burned child first experiences a decrease in cardiac output with a subsequent hypermetabolic response during which cardiac output increases dramatically. During this heightened metabolic state, the child is a risk for insulin resistance and increased protein catabolism.

When a child enters the acute care setting or clinic following a burn injury, for which of the following immunizations is it important to check for the last booster? a) Pertussis b) Tetanus c) Meningitis d) Diphtheria

b)Tetanus Explanation: For any burn, check the child's tetanus immunization status on admission and ensure that tetanus toxoid is given if the child's immunizations are not up to date, because anaerobic and aerobic bacteria can grow at the interface between burned and healthy tissue. Pertussis, diphtheria and meningitis are not important immunizations to check for with a burn patient.

A 6-year-old child is diagnosed with tinea capitis and treatment is initiated. The nurse instructs the parents to have the child return to school within which time frame? a) 5 days b) 72 hours c) 1 week d) 24 hours

c)1 week Explanation: Once treatment is initiated for tinea capitis, the child can return to school or day care after 1 week.

A school-age child is brought to the office of the camp nurse with a small, superficial burn. Which of the following actions by the nurse would be the most appropriate action for the nurse to do first? a) Apply a topical anesthetic ointment b) Administer acetaminophen c) Apply cold compresses to the area d) Cover the area with a sterile bandage

c)Apply cold compresses to the area Explanation: Cool water is an excellent emergency treatment for burns involving small areas. The immediate application of cool compresses or cool water to burn areas appears to inhibit capillary permeability and thus suppress edema, blister formation, and tissue destruction.

Which of the following accurately depicts the hemodynamic changes that occur in the body within the first 24 to 48 hours after a burn? a) Hematocrit increases and WBC count decreases b) Hematocrit and WBC counts decrease c) Hematocrit and WBC counts elevate d) Hemoglobin and WBC counts decrease

c)Hematocrit and WBC counts elevate Explanation: In the first 24 to 48 hours after a burn, the hematocrit will often be elevated secondary to fluid loss and the WBC may also be elevated as an acute-phase reaction, which later could indicate infection.

The nurse is developing the plan of care for a 3-year-old child diagnosed with atopic dermatitis. When reviewing the desired patient outcomes which of the following are common focuses for a child with this diagnosis? Select all that apply. a) Maintenance of skin integrity b) Prevention of infection c) Promotion of skin hydration d) Pain management e) Reduction in anxiety

c)Promotion of skin hydration a)Maintenance of skin integrity b)Prevention of infection Explanation: When caring for the child with atopic dermatitis the focus of care will be on the prevention of infection, maintenance of skin integrity, and promotion of skin hydration.

The nurse is caring for an infant who has impetigo and is hospitalized. Which of the following nursing interventions is the highest priority for this child? a) The nurse applies elbow restraints to the infant. b) The nurse applies topical antibiotics to the lesions. c) The nurse follows contact precautions. d) The nurse soaks the skin with warm water.

c)The nurse follows contact precautions. Explanation: Impetigo is highly contagious and can spread quickly. The nurse should follow contact (skin and wound) precautions, including wearing a cover gown and gloves. The nurse will soak the crusts with warm water, apply topical antibiotics, and apply elbow restraints, but these are not as high a priority as trying to prevent the spread of the infection by following contact precautions.

A nurse providing teaching on ways to promote skin hydration for the parents of an infant with atopic dermatitis. Which of the following responses indicates a need for further teaching? a) "We need to avoid any skin product containing perfumes, dyes, or fragrances." b) "We should use soap to clean only dirty areas." c) "We should use a mild soap for sensitive skin." d) "We should bathe our child in hot water, twice a day."

d)"We should bathe our child in hot water, twice a day." Explanation: The nurse should emphasize that the parents should avoid hot water. The child should be bathed twice a day in warm water. The other statements are correct.

A nurse is caring for a child with tinea pedis. Which of the following assessment findings would the nurse expect to note? a) Inflamed boggy mass filled with pustules b) Erythema, scaling, maceration in the inguinal creases and inner thighs c) Patches of scaling in the scalp with central hair loss d) Red scaling rash on soles and between the toes

d)Red scaling rash on soles and between the toes Explanation: Tinea pedis presents with red scaling rash on soles, and between the toes. Tinea capitis presents with patches of scaling in the scalp with central hair loss and the risk of kerion development (inflamed boggy mass filled with pustules). Tinea cruris presents with erythema, scaling, maceration in the inguinal creases and inner thighs.

21. With gastroesophageal reflux symptoms, the purpose of esophageal pH monitoring is evaluating for: esophagitis from the reflux. frequency of the reflux. gastric emptying delays. possible obstructions.

frequency of the reflux.

The nurse interviewing parents of an infant with pyloric stenosis would expect the parents to report if the infant has had

projectile vomiting

14. The nurse is caring for a 1-year-old who has just been diagnosed with viral encephalitis. The parents ask if their child will be admitted to the hospital. Select the nurse's best response. 1. "Your child will likely be sent home because encephalitis is usually caused by a virus and not bacteria." 2. "Your child will likely be admitted to the pediatric floor for intravenous antibiotics and observation." 3. "Your child will likely be admitted to the PICU for close monitoring and observation." 4. "Your child will likely be sent home because she is only 1 year old. We see

1. Although encephalitis is usually caused by a viral infection, the child is usually admitted for close observation. 2. Intravenous antibiotics are not given to the child with viral encephalitis. 3. CORRECT The young child with encephalitis should be admitted to a PICU where close observation and monitoring are available. The child should be observed for signs of increased ICP and for cardiac and respiratory compromise. 4. The child would not be discharged as observation for complications is necessary. As a general rule, younger children tend to have more complications and require a PICU admission.

37. The parent of a toddler newly diagnosed with cerebral palsy (CP) asks the nurse what caused it. The nurse should answer with which of the following? 1. Most cases are caused by unknown prenatal factors. 2. It is commonly caused by perinatal factors. 3. The exact cause is not known. 4. The exact cause is known in every instance.

1. Most cases are caused by unknown prenatal factors.

41. The nurse is working in a school health clinic, and a teen mentions that her older sister just had a baby born with a myelomeningocele. The teen is wondering if there is anything she can do to prevent this from happening to her baby when she decides to have children. Which is the best response? 1. Take a multivitamin with folic acid daily. 2. Eat more fruits and vegetables daily. 3. Have breakfast every morning. 4. There is nothing that can be done to decrease the risk.

1. Take a multivitamin with folic acid daily

50. A child with cerebral palsy (CP) has been fitted for braces and is beginning physical therapy to assist with ambulation. The parents ask why he needs the braces when he was crawling without any assistive devices. Select the nurse's best response. 1. "The CP has progressed, and he now needs more assistance to ambulate." 2. "As your child grows, different muscle groups may need more assistance." 3. "Most children with CP need braces to help with ambulation." 4. "We have found that when children with CP use braces, they are less likely to fall."

2. "As your child grows, different muscle groups may need more assistance."

46. Which child requires continued follow-up because of behaviors suspicious of cerebral palsy (CP)? 1. 1-month-old who demonstrates the startle reflex when a loud noise is heard. 2. 6-month-old who always reaches for toys with the right hand. 3. 14-month-old who has not begun to walk. 4. 2-year-old who has not yet achieved bladder control during waking hours.

2. 6-month-old who always reaches for toys with the right hand.

A 4-month-old has had vomiting and diarrhea for 24 hours. The infant is fussy, and the anterior fontanel is sunken. The nurse notes the infant does not produce tears when crying. Which task will help confirm the diagnosis of dehydration? 1. Urinalysis obtained by bagged specimen. 2. Urinalysis obtained by sterile catheterization. 3. Analysis of serum electrolytes. 4. Analysis of cerebrospinal fluid.

3. Analysis of serum electrolytes.

98. Which applies to cerebral palsy? Select all that apply. 1. It is the most common chronic disorder of childhood. 2. Hyperbilirubinemia increases the risk of cerebral palsy. 3. It is a progressive chronic disorder. 4. Most children do not experience any learning disabilities. 5. There is a familial tendency seen in children with cerebral palsy.

1, 2

89. The nurse is doing discharge teaching for a 3-month-old with a new shunt placed for hydrocephalus. Which are signs and symptoms of hydrocephalus that the parents may see if the shunt malfunctions? Select all that apply. 1. Vomiting. 2. Irritability. 3. Poor feeding. 4. Headache. 5. Sunken fontanel. 6. Seizures. 7. Inability to wake up infant. 8. Hyperactivity.

1, 2, 3, 6, 7

13. Which of the following can be manifestations of leukemia in a child? Select all that apply. 1. Leg pain. 2. Fever. 3. Excessive weight gain. 4. Bruising. 5. Enlarged lymph nodes.

1,2,4,5

59. Which is assessed to diagnose pediculosis capitis? Select all that apply. 1. Crawling insects. 2. White flakes in the hair. 3. Nits attached close to scalp. 4. Inflammatory papules. 5. Dark brown hair.

1,3,4

43. A child is admitted to the pediatric unit with spastic CP. Which would the nurse expect a child with spastic CP to demonstrate? Select all that apply. 1. Increased deep tendon reflexes. 2. Decreased muscle tone. 3. Scoliosis. 4. Contractures. 5. Scissoring. 6. Good control of posture. 7. Good fine motor skills.

1,3,4,5

45. Which of the following best describes the action of chemotherapeutic agents used in the treatment of cancer in children? 1. Suppress the function of normal lymphocytes in the immune system. 2. Are alkylating agents and are cell-specific. 3. Cause a replication of DNA and are cell-specific. 4. Interrupt cell cycle, thereby causing cell death.

1. All chemotherapy is immunosuppressive as most childhood cancers affect the immune system.

10. The nurse is caring for a child with leukemia. The nurse should be aware that children being treated for leukemia may experience which of the following complications? Select all that apply. 1. Anemia. 2. Infection. 3. Bleeding tendencies. 4. Bone deformities. 5. Polycythemia.

1. Anemia is caused by decreased production of red blood cells. 2. Infection risk in leukemia is secondary to the neutropenia.

13. The clinical manifestations of minimal change nephrotic syndrome (MCNS) are due to which of the following? 1. Chemical changes in the composition of albumin. 2. Increased permeability of the glomeruli. 3. Obstruction of the capillaries of the glomeruli. 4. Loss of the kidney's ability to excrete waste and concentrate urine.

13. 1. Albumin does not undergo any chemical changes in MCNS. 2. CORRECT Increased permeability of the glomeruli in MCNS allows large substances such as protein to pass through and be excreted in the urine. 3. Obstruction of the capillaries of the glomeruli due to the formation of antibody-antigen complexes occurs in glomerulonephritis. 4. Loss of the kidneys' ability to excrete waste and concentrate urine occurs in renal failure.

19. A 13-month-old is discharged following repair of his epispadias. Which statement made by the parents indicates they understand the discharge teaching? 1. "If a mucous plug forms in the urinary drainage tube, we will irrigate it gently to prevent a blockage." 2. "If a mucous plug forms in the urinary drainage tube, we will allow it to pass on its own because this is a sign of healing." 3. "We will make sure the dressing is loosely applied to increase the toddler's comfort." 4. "If we notice any yellow drainage, we will know that everything is healing well."

19. 1. CORRECT Any mucous plugs should be removed by irrigation to prevent blockage of the urinary drainage system. 2. The mucous plug should be removed by irrigation because it could cause a blockage of the urinary drainage system. 3. The dressing is usually a compression type of dressing that helps decrease edema. 4. Foul-smelling yellow drainage is often a sign of infection that needs to be evaluated.

21. The diet that produces anticonvulsant effects from ketosis consists of: 1. High-fat and low-carbohydrate foods. 2. High-fat and high-carbohydrate foods. 3. Low-fat and low-carbohydrate foods. 4. Low-fat and high-carbohydrate foods.

21. 1. CORRECT High fat and low carbohydrates are the components of the ketogenic diet. 2. High fat and high carbohydrates are the components of the ketogenic diet. 3. Low fat and low carbohydrates are the components of the ketogenic diet. 4. Low fat and high carbohydrates are the components of the ketogenic diet.

30. A child in the PICU with a head injury is comatose and unresponsive. The parent asks if he needs pain medication. Select the nurse's best response. 1. "Pain medication is not necessary as he is unresponsive and cannot feel pain." 2. "Pain medication may interfere with his ability to respond and may mask any signs of improvement." 3. "Pain medication is necessary to make him comfortable." 4. "Pain medication is necessary for comfort, but we use it cautiously as it increases the demand for oxygen."

30. 1. Even if the child is unresponsive, the child can still feel pain. 2. If pain medication is administered cautiously, the child can still be monitored, and signs of improvement will be evident. 3. CORRECT Pain medication promotes comfort and ultimately decreases ICP. 4. Pain medication decreases the demand for oxygen.

53. The nurse is caring for a child with cerebral palsy (CP) whose weight is in the fifth percentile and who has been hospitalized for aspiration pneumonia. His parents are anxious and state that they do not want a G-tube placed. Which would be the nurse's best response? 1. "A G-tube will help your son gain weight and reduce his risk for future hospitalizations due to pneumonia." 2. "G-tubes are very easy to care for and will make feeding time easier for your family." 3. "Are you concerned that you will not be able to care for his G-tube?" 4. "Tell me your thoughts about G-tubes."

4. "Tell me your thoughts about G-tubes."

39. Which is included in the plan of care for a newborn who has a myelomeningocele? 1. Place the child in the prone position with a sterile dry dressing over the defect. Slowly begin oral gastric feeds to prevent the development of necrotizing enterocolitis. 2. Place the child in the prone position with a sterile dry dressing over the defect. Begin intravenous fluids to prevent dehydration. 3. Place the child in the prone position with a sterile moist dressing over the defect. Slowly begin oral gastric feeds to prevent the development of necrotizing enterocolitis. 4. Place the child in the prone position with a sterile moist dressing over the defect. Begin intravenous fluids to prevent dehydration.

4. Place the child in the prone position with a sterile moist dressing over the defect. Begin intravenous fluids to prevent dehydration.

1. The nurse explains that a ventricular septal defect will allow: a. blood to shunt left to right, causing increased pulmonary flow and no cyanosis. b. blood to shunt right to left, causing decreased pulmonary flow and cyanosis. c. no shunting because of high pressure in the left ventricle. d. increased pressure in the left atrium, impeding circulation of oxygenated blood in the circulating volume.

ANS: A Pulmonary blood flow is increased when a ventricular septal defect exists. The blood shifts from left to right because of the higher pressure in the left ventricle. This particular shift does not cause cyanosis.

The most important nursing intervention when caring for a child with myelomeningocele in the preoperative stage is which of the following? a. Take vital signs every hour. b. Place child on side to decrease pressure on the spinal sac. c. Watch for signs that might indicate developing hydrocephalus. d. Apply a heat lamp to facilitate drying and toughening of the sac.

ANS: B The spinal sac is protected from damage until surgery is performed. Early surgical closure is recommended to prevent local trauma and infection. Monitoring vital signs and watching for signs that might indicate developing hydrocephalus are important interventions, but preventing trauma to the sac is a priority. The sac is kept moist until surgical intervention is done.

7. Which child is at risk for developing glomerulonephritis? 1. A 3-year-old who had impetigo 1 week ago. 2. A 5-year-old with a history of five UTIs in the previous year. 3. A 6-year-old with new-onset type 1 diabetes. 4. A 10-year-old recovering from viral pneumonia

1. CORRECT Impetigo is a skin infection caused by the streptococcal organism that is commonly associated with glomerulonephritis. 2. Frequent UTIs have not been associated with glomerulonephritis. 3. Type 1 diabetes is not a cause of glomerulonephritis. 4. Glomerulonephritis can be caused by a streptococcal organism, not a viral pneumonia.

12. Which statement by a parent is most consistent with minimal change nephrotic syndrome (MCNS)? 1. "My child missed 2 days of school last week because of a really bad cold." 2. "After camping last week, my child's legs were covered in bug bites." 3. "My child came home from school a week ago due to vomiting and stomach cramps." 4. "We have a pet turtle but no one washes their hands after playing with the turtle."

12. 1. CORRECT An upper respiratory infection often precedes MCNS by a few days. 2. Bug bites are not typically associated with MCNS. 3. Vomiting and abdominal cramping are not typically associated with MCNS. 4. Pet turtles often carry salmonella, which can cause vomiting and diarrhea but not MCNS.

15. A child with minimal change nephrotic syndrome (MCNS) has generalized edema. The skin appears stretched, and areas of breakdown are noted over the bony prominences. The child has been receiving Lasix twice daily for several days. Which does the nurse expect to be included in the treatment plan to reduce edema? 1. An increase in the amount and frequency of Lasix. 2. Addition of a second diuretic, such as mannitol. 3. Administration of intravenous albumin. 4. Elimination of all fluids and sodium from the child's diet.

15. 1. The dosage of the diuretic may be adjusted, but other medications such as albumin are likely to be used. 2. Mannitol is not usually used in the treatment of MCNS. 3. CORRECT In cases of severe edema, albumin is used to help return the fluid to the bloodstream from the subcutaneous tissue. 4. Although sodium and fluids are restricted in the severely edematous child, they are not eliminated completely.

37. The nurse is caring for an infant with a myelomeningocele. The parents ask the nurse why the nurse keeps measuring the baby's head circumference. Select the nurse's best response: 1. "Babies' heads are measured to ensure growth is on track." 2. "Babies with a myelomeningocele are at risk for hydrocephalus, which shows up as an increase in head size." 3. "Because your baby has an opening on the spinal cord, your infant is at risk for meningitis, which can show up as an increase in head size." 4. "Many infants with myelomeningocele have microcephaly, which can show up as a decrease in head size."

2. "Babies with a myelomeningocele are at risk for hydrocephalus, which shows up as an increase in head size."

50. The nurse is providing discharge instructions to the parents of an infant who has had surgery to open a low imperforate anus. The nurse knows that the discharge instructions have been understood when the child's parents say: 1. "We will use an oral thermometer because we cannot use a rectal one." 2. "We will call the physician if the stools change in consistency." 3. "Our infant will never be toilet-trained." 4. "We understand that it is not unusual for our infant's urine to contain stool."

2. "We will call the physician if the stools change in consistency."

21. The manifestations of hemolytic uremic syndrome (HUS) are due primarily to which event? 1. The swollen lining of the small blood vessels damages the red blood cells, which are then removed by the spleen, leading to anemia. 2. There is a disturbance of the glomerular basement membrane, allowing large proteins to pass through. 3. The red blood cell changes shape, causing it to obstruct microcirculation. 4. There is a depression in the production of all formed elements of the blood

21. 1. CORRECT The swollen lining of the small blood vessels damages the red blood cells, which are then removed by the spleen. 2. The increased permeability of the basement membrane occurs in MCNS. 3. The red blood cell changing shape is typical of sickle cell anemia. 4. The depression of all formed elements of the blood occurs in aplastic anemia.

5. Which signs best indicate increased intracranial pressure (ICP) in an infant? Select all that apply. 1. Sunken anterior fontanel. 2. Complaints of blurred vision. 3. High-pitched cry. 4. Increased appetite. 5. Sleeping more than usual.

3, 5. 1. The anterior fontanel is usually raised and bulging in infants with increased ICP. 2. The infant is not able to comprehend blurred vision or make any statements. 3. A high-pitched cry is often indicative of increased ICP in infants. 4. The infant with increased ICP usually has a poor appetite and does not feed well. 5. The infant may be sleeping more than usual due to increased ICP.

17. The nurse is caring for a newborn with hypospadias. His parents ask if circumcision is an option. Which is the nurse's best response? 1. "Circumcision is a fading practice and is now contraindicated in most children." 2. "Circumcision in children with hypospadias is recommended because it helps prevent infection." 3. "Circumcision is an option, but it cannot be done at this time." 4. "Circumcision can never be performed in a child with hypospadias."

3. "Circumcision is an option, but it cannot be done at this time." Because you have to wait for hypospadias to be fixed

15. The nurse knows further education is needed about Reye syndrome when a mother states: 1. "I will have my children immunized against varicella and influenza." 2. "I will make sure not to give my child any products containing aspirin." 3. "I will give aspirin to my child to treat a headache." 4. "Children with Reye syndrome are admitted to the hospital."

3. "I will give aspirin to my child to treat a headache"

40. The parent of a young child with CP brings the child to the clinic for a checkup. Which parent's statement indicates an understanding of the child's long-term needs? 1. "My child will need all my attention for the next 10 years." 2. "Once in school, my child will catch up and be like the other children." 3. "My child will grow up and need to learn to do things independently." 4. "I'm the one who knows the most about my child and can do the most for my child."

3. "My child will grow up and need to learn to do things independently."

44. The nurse is caring for an infant with pyloric stenosis. The parent asks if any future children will likely have pyloric stenosis. Select the nurse's best response. 1. "You seem worried; would you like to discuss your concerns?" 2. "It is very rare for a family to have more than one child with pyloric stenosis." 3. "Pyloric stenosis can run in families. It is more common among males." 4. "Although there can be a genetic link, it is very unusual for girls to have pyloric stenosis."

3. "Pyloric stenosis can run in families. It is more common among males."

25. A 5-year-old is discharged from the hospital following the diagnosis of hemolytic uremic syndrome (HUS). The child has been free of diarrhea for 1 week, and renal function has returned. The parent asks the nurse when the child can return to school. Which is the nurse's best response? 1. "Immediately, as your child is no longer contagious." 2. "It would be best to keep your child home for a few more weeks because the immune system is weak, and there could be a relapse of HUS." 3. "Your child will be contagious for approximately another 10 days, so it is best to not allow a return just yet." 4. "It would be best to keep your child home to monitor urinary output."

3. "Your child will be contagious for approximately another 10 days, so it is best to not allow a return just yet."

3. A 4-month-old has had vomiting and diarrhea for 24 hours. The infant is fussy, and the anterior fontanel is sunken. The nurse notes the infant does not produce tears when crying. Which task will help confirm the diagnosis of dehydration? 1. Urinalysis obtained by bagged specimen. 2. Urinalysis obtained by sterile catheterization. 3. Analysis of serum electrolytes. 4. Analysis of cerebrospinal fluid.

3. Analysis of serum electrolytes

83. Which should the nurse teach the parent of a child with suspected meningitis? 1. Antibiotics are not initiated until the cerebrospinal fluid cultures are definitive for specificity to prevent resistance. 2. Antibiotics are useless against viral infections, so they are not used for meningitis. 3. Antibiotics should be started before the cerebrospinal fluid cultures are definitive; culture results may take up to 3 days. 4. Antibiotic initiation is based on the age, signs, and symptoms of the child, not on the causative agent.

3. Antibiotics should be started before the cerebrospinal fluid cultures are definitive; culture results may take up to 3 days.

41. Which does the nurse include in a child with myelomeningocele postoperative plan of care following ligament release? 1. Encourage the child to resume a regular diet, beginning slowly with bland foods that are easily digested, such as bananas. 2. Encourage the child to blow balloons to increase deep breathing and avoid postoperative pneumonia. 3. Assist the child to change positions to avoid skin breakdown. 4. Provide education on dietary requirements to prevent obesity and skin breakdown.

3. Assist the child to change positions to avoid skin breakdown.

The nurse is caring for a 9-month-old with diarrhea secondary to rotavirus. The child has not vomited and is mildly dehydrated. Which is likely to be included in the discharge teaching? 1. Administer Imodium as needed. 2. Administer Kaopectate as needed. 3. Continue breastfeeding per routine. 4. The infant may return to day care 24 hours after antibiotics have been started.

3. Breastfeeding is usually well tolerated and helps prevent death of intestinal villi and malabsorption.

34. A newborn with a repaired myelomeningocele is assessed for hydrocephalus. Which would the nurse expect in an infant with hydrocephalus? 1. Low-pitched cry and depressed fontanel. 2. Low-pitched cry and bulging fontanel. 3. Bulging fontanel and downwardly rotated eyes. 4. Depressed fontanel and upwardly rotated eyes.

3. Bulging fontanel and downwardly rotated eyes.

36. The nurse is caring for a 4-month-old who has just had an isolated cleft lip repaired. Select the best position for the child in the immediate post-operative period. 1. Right side-lying. 2. Left side-lying. 3. Supine. 4. Prone.

3. Supine

19. The nurse is caring for a 4-month-old with gastroesophageal reflux (GER). The infant is due to receive Zantac (rantadine). Based on the medication's mechanism of action, when should this medication be administered? 1. Immediately before a feeding. 2. 30 minutes after the feeding. 3. 30 minutes before the feeding. 4. At bedtime.

3. Zantac decreases gastric acid secretion and should be administered 30 minutes before a feeding.

37. A child with leukemia is receiving chemotherapy and is complaining of nausea. The nurse has been giving the scheduled antiemetic. Which of the following should the nurse do when the child is nauseated? 1. Encourage low-protein foods. 2. Encourage low-caloric foods. 3. Offer the child's favorite foods. 4. Offer cool, clear liquids.

4. Offer cool, clear liquids. They are better tolerated

6. Which is the best way to obtain a urine sample in an 8-month-old being evaluated for a urinary tract infection (UTI)? 1. Carefully cleanse the perineum from front to back, and apply a self-adhesive urine collection bag to the perineum. 2. Insert an indwelling Foley catheter, obtain the sample, and wait for results. 3. Place a sterile cotton ball in the diaper, and immediately obtain the sample with a syringe after the first void. 4. Using a straight catheter, obtain the sample, and immediately remove the catheter without waiting for the results of the urine sample.

6. 1. A sample obtained from a urine bag would contain microorganisms from the skin, causing contamination of the sample. 2. There is no need to leave the catheter in because it serves as a portal for infection. 3. The cotton ball would not remain sterile and would therefore contaminate the urine sample. 4. CORRECT An in-and-out catheterization is the best way to obtain a urine culture in a child who is not yet toilet-trained.

The nurse is caring for a 2-year-old boy with a burn. Which of the following findings would warrant referral to a burn unit? a) The boy has a superficial burn on his chest. b) The boy has a superficial burn on his hands. c) The boy has suffered a chemical burn. d) The boy has a first-degree burn on the upper arm.

c)The boy has suffered a chemical burn. Explanation: According to the Committee on Trauma of the American College of Surgeons, chemical burns warrant referral to a burn unit. A partial-thickness burn greater than 10% of the body surface area would warrant a referral to a burn unit. A superficial burn on the chest or hands does not warrant a referral to a burn unit. A first-degree burn would most likely be classified as a superficial burn, which would not warrant a referral to a burn unit.

26. Which should the nurse do first when caring for an infant who just had a repair of a myelomeningocele? 1. Weigh diapers for 24-hour urine output. 2. Measure head circumference. 3. Offer clear fluids. 4. Assess for infection.

2. Measure head circumference

19. A child with a seizure disorder has been having episodes during which she drops her pencil and simply appears to be daydreaming. This is most likely a/an: 1. Absence seizure. 2. Akinetic seizure. 3. Non-epileptic seizure. 4. Simple spasm seizure.

1. CORRECT Absence seizures occur frequently and last less than 30 seconds. The child experiences a brief loss of consciousness during which she may have a change in activity. These children rarely fall, but they may drop an object. The condition is often confused with daydreaming. 2. Akinetic seizures occur when the young child experiences a brief loss of consciousness and postural tone and falls to the ground. The child quickly regains consciousness. 3. A non-epileptic seizure is a seizure that occurs secondary to another disorder, such as a fever or increased ICP. 4. A simple spasm seizure is not a diagnosis.

Which nursing intervention would be of highest priority for a 2-year-old suspected of ingestion digoxin? 1. Provide supplemental oxygen. 2. Establish intravenous access. 3. Draw blood for a STAT digoxin level. 4. Provide continuous cardiac monitoring.

1. Continuous cardiac monitoring would be the priority because of the bradycardia and dysrhythmias that can occur with digoxin toxicity. Oxygen may be needed if there is enough bradycardia causing a decrease in oxygen saturation. 2. The priority is to establish continuous cardiac monitoring. If it is determined that venous access is necessary, then that can be established. 3. The digoxin level would be good to know, but that is not the priority. 4. CORRECT Bradycardia and cardiac dysrhythmias are common signs of digoxin toxicity in children. Continuous cardiac monitoring is the highest priority to detect dysrhythmias before they became lethal.

12. Select the best room assignment for a newly admitted child with bacterial meningitis. 1. Semiprivate room with a roommate who also has bacterial meningitis. 2. Semiprivate room with a roommate who has bacterial meningitis but has received intravenous antibiotics for more than 24 hours. 3. Private room that is dark and quiet with minimal stimulation. 4. Private room that is bright and colorful and has developmentally appropriate activities available.

1. The child with bacterial meningitis should be placed in a private room isolated from all other patients. Bacterial meningitis is caused by many pathogens, and patients should be isolated from each other. 2. The child with bacterial meningitis should be placed in a private room isolated from all other patients. Bacterial meningitis is caused by many pathogens, and patients should be isolated from each other. 3. CORRECT A quiet private room with minimal stimulation is ideal as the child with meningitis should be in a quiet environment to avoid cerebral irritation. 4. A bright room with developmental activities may cause irritation and increase ICP.

16. The nurse is caring for an 8-week-old male who has just been diagnosed with Hirschsprung disease. The parents ask what they should expect. Select the nurse's best response. 1. "It is really an easy disease to manage. Most children are placed on stool softeners to help with constipation until it resolves." 2. "A permanent stool diversion, called a colostomy, will be placed by the surgeon to bypass the narrowed area." 3. "Daily bowel irrigations will help your child maintain regular bowel habits." 4. "Although your child will require surgery, there are different ways to manage the disease, depending on how much bowel is involved."

16. 1. The constipation will not resolve with stool softeners. The affected bowel needs to be removed. 2. Most colostomies are not permanent. The large intestine is usually reattached, and the colostomy is taken down. 3. The child with Hirschsprung disease requires surgery to remove the aganglionic portion of the large intestine. 4. CORRECT The aganglionic portion needs to be removed. Although most children have a temporary colostomy placed, many infants are able to bypass the colostomy and have the bowel immediately reattached.

20. The nurse is administering Prilosec (omeprazole) to a 3-month-old with gastro - esophageal reflux (GER). The child's parents ask the nurse how the medication works. Select the nurse's best response. 1. "Prilosec is a proton pump inhibitor that is commonly used for reflux in infants." 2. "Prilosec decreases stomach acid, so it will not be as irritating when your child spits up." 3. "Prilosec helps food move through the stomach quicker, so there will be less chance for reflux." 4. "Prilosec relaxes the pressure of the lower esophageal sphincter."

2. "Prilosec decreases stomach acid, so it will not be as irritating when your child spits up."

22. Which medication should the nurse anticipate administering first to a child in status epilepticus? 1. Establish an intravenous line, and administer intravenous lorazepam. 2. Administer rectal diazepam. 3. Administer an oral glucose gel to the side of the child's mouth. 4. Administer oral diazepam.

22. 1. It is very difficult and time consuming to establish an intravenous line on a child who is experiencing a generalized seizure. Rectal diazepam is first administered in an attempt to stop the seizure long enough to establish a line, and then medication is administered intravenously. 2. CORRECT Rectal diazepam is first administered in an attempt to stop the seizure long enough to establish an IV, and then IV medication is administered. 3. Although the child may become hypoglycemic due to increased metabolic demands, stopping the seizure with rectal diazepam is the first priority. Medication is not placed in the mouth of a child experiencing a generalized seizure as it increases the risk of injury and aspiration. 4. Stopping the seizure with rectal diazepam takes priority. Nothing should be administered orally to a patient who is unconscious.

31. The nurse is caring for a child with a skull fracture who is unconscious and has severely increased intracranial pressure (ICP). The nurse notes the child's temperature to be 104°F (40°C). Which should the nurse do first? 1. Place a cooling blanket on the child. 2. Administer Tylenol (acetaminophen) via nasogastric tube. 3. Administer Tylenol (acetaminophen) rectally. 4. Place ice packs in the child's axillary areas.

1. Place a cooling blanket on the child

4. An 18-month-old is discharged from the hospital after having a febrile seizure secondary to exanthem subitum (roseola). On discharge, the mother asks the nurse if her 6-year-old twins will get sick. Which teaching about the transmission of roseola would be most accurate? 1. The child should be isolated in the home until the vesicles have dried. 2. The child does not need to be isolated from the older siblings. 3. Administer acetaminophen to the older siblings to prevent seizures. 4. Monitor older children for seizure development.

1. The rash is pink and maculopapular, not vesicular. The incubation period is 5 to 15 days and more commonly seen in children 6 months to 3 years of age. Isolating the siblings is not necessary. 2. CORRECT Roseola transmission is unknown and more commonly seen in children 6 months to 3 years of age, so siblings do not need to be isolated. 3. Because the siblings have no history of seizures, it is not necessary to administer acetaminophen to prevent seizures. 4. Febrile seizures are not usually seen in children older than 6 years, and because they have no history of seizures, it is not necessary to monitor them for seizure development.

Which clinical assessment of a neonate with bacterial meningitis would warrant immediate intervention? 1. Irritability. 2. Rectal temperature of 100.6°F (38.1°C). 3. Quieter than usual. 4. Respiratory rate of 24 breaths per minute.

1. With the diagnosis of suspected bacterial meningitis, the neonate is expected to be irritable, which frequently accompanies increased intracranial pressure. 2. A rectal temperature of 38.1°C or 100.6°F indicates a low-grade fever and is not as concerning as the slower-than-normal respiratory rate of 24. 3. The fact the infant is quieter than normal is in response to the slow respiratory rate and sepsis the neonate is experiencing. 4. CORRECT A normal neonate's respiratory rate is 30 to 60 breaths per minute. Neonates' respiratory systems are immature, and the rate may initially double in response to illness. If no immediate interventions are begun when there is respiratory distress, a neonate's respiratory rate will slow down, develop worsening respiratory distress, and, eventually, respiratory arrest. Neonates with slower or faster respiratory rates are true emergency cases; they require identification of the cause of distress.

37. Which should be included in the plan of care for a 14-month-old whose cleft palate was repaired 12 hours ago? Select all that apply. 1. Allow the infant to have familiar items of comfort such as a favorite stuffed animal and a "sippy" cup. 2. Once liquids have been tolerated, encourage a bland diet such as soup, Jell-O, and saltine crackers. 3. Administer pain medication on a regular schedule, as opposed to an as-needed schedule. 4. Use a Yankauer suction catheter on the infant's mouth to decrease the risk of aspiration of oral secretions. 5. When discharged, remove elbow restraints.

1. Allow the infant to have familiar items of comfort such as a favorite stuffed animal and a "sippy" cup. 3. Administer pain medication on a regular schedule, as opposed to an as-needed schedule.

31. The nurse is caring for a child with a diagnosis of ALL who is receiving chemotherapy. The nurse notes that the child's platelet count is 20,000/mm3. Based on this laboratory finding, what information should the nurse provide to the child and parents? 1. A soft toothbrush should be used for mouth care. 2. Isolation precautions should be started immediately. 3. The child's vital signs, including blood pressure, should be monitored every 4 hours. 4. All visitors should be discouraged from coming to see the family.

1. Because the platelet count is decreased, there is a significant risk of bleeding, especially in soft tissue. The use of the soft toothbrush should help prevent bleeding of the gums.

21. Which priority item should be placed at the bedside of a newborn with myelomeningocele? 1. A bottle of normal saline. 2. A rectal thermometer. 3. Extra blankets. 4. A blood pressure cuff.

1. Before the surgical closure of the sac, the infant is at risk for infection. A sterile dressing is placed over the sac to keep it moist and help prevent it from tearing.

11. The nurse is caring for a 6-month-old infant diagnosed with meningitis. When the child is placed in the supine position and flexes his neck, the nurse notes he flexes his knees and hips. This is referred to as: 1. Brudzinski sign. 2. Cushing triad. 3. Kernig sign. 4. Nuchal rigidity.

1. CORRECT Brudzinski sign occurs when the child responds to a flexed neck with an involuntary flexion of the hips and/or knees. 2. Cushing triad is a sign of increased ICP and is manifested with an increase in systolic blood pressure, decreased heart rate, and irregular respirations. 3. Kernig sign occurs when there is resistance or pain in response to raising the child's flexed leg. 4. Nuchal rigidity occurs when there is a resistance to neck flexion.

4. Which should the nurse teach a group of girls and parents about the importance of preventing urinary tract infections (UTIs)? 1. Avoiding constipation has no effect on the occurrence of UTIs. 2. After urinating, always wipe from back to front to prevent fecal contamination. 3. Hygiene is an important preventive measure and can be accomplished with frequent tub baths. 4. Increasing fluids will help prevent and treat UTIs

1. The increased pressure associated with evacuating the hardened stool can result in the backflow of urine into the bladder, leading to infection. 2. To prevent infection, a female child should wipe from front to back. 3. Tub baths are not recommended because they may cause irritation of the urethra, leading to infection. 4. CORRECT Increasing fluids will help flush the bladder of any organism, encourage urination, and prevent stasis of urine.

16. When assessing the neurological status of an 8-month-old, the nurse should check for which of the following? 1. Clarity of speech. 2. Interaction with staff. 3. Vision test. 4. Romberg test.

1. The infant is preverbal, so assessing clarity of speech is not age-appropriate or developmentally appropriate. 2. CORRECT Assessment for alteration in developmentally expected behaviors, such as stranger anxiety, is helpful. Interaction with staff is not to be expected due to stranger anxiety. 3. Vision testing is not the priority assessment. 4. An 8-month-old infant is not tested with the Romberg test.

42. Which of the following is correct regarding prognostic factors for determining survival for a child newly diagnosed with ALL? 1. The initial white blood cell count on diagnosis. 2. The race of the child. 3. The amount of time needed to initiate treatment. 4. The allergy history of the child.

1. The initial white blood cell count on diagnosis.

7. A child with a ventriculoperitoneal (VP) shunt complains of headache and blurry vision and now experiences irritability and sleeping more than usual. The parents ask the nurse what they should do. Select the nurse's best response. 1. "Give her some acetaminophen, and see if her symptoms improve. If they do not improve, bring her to the pediatrician's office." 2. "It is common for girls to have these symptoms, especially prior to beginning their menstrual cycle. Give her a few days, and see if she improves." 3. "You are probably worried that she is having a problem with her shunt. This is very unlikely as it has been working well for 9 years." 4. "You should immediately take her to the emergency room as these may be symptoms of a shunt malfunction."

1. These are symptoms of a shunt malfunction and should be evaluated immediately. 2. Although these symptoms may be associated with the start of a girl's menstrual cycle, they are symptoms of a shunt malfunction and require immediate evaluation. 3. A shunt can malfunction at any point and should be evaluated when signs of increased ICP are evident. 4. CORRECT. These are symptoms of a shunt malfunction and should be evaluated immediately. \

22. The nurse is caring for an infant with myelomeningocele who is going to surgery later today for closure of the sac. Which would be a priority nursing diagnosis before surgery? 1. Alteration in parent-infant bonding. 2. Altered growth and development. 3. Risk of infection. 4. Risk for weight loss.

1. This is certainly a possibility, but in the pre-operative period risk of infection is the priority. 2. Altered growth and development may occur, but in the pre-operative period risk of infection is the priority. 3. CORRECT A normal saline dressing is placed over the sac to prevent tearing, which would allow the cerebrospinal fluid to escape and microorganisms to enter and cause an infection. 4. It is normal in the first 2 weeks of life to lose up to 10% of birth weight. In fact, this infant may lose more weight because of surgery, but the priority is risk of infection.

13. The nurse is caring for an infant diagnosed with Hirschsprung disease. The mother states she is pregnant with a boy and wants to know if her new baby will likely have the disorder. Select the nurse's best response. 1. "Genetics play a small role in Hirschsprung disease, so there is a chance the baby will develop it as well." 2. "There is no evidence to support a genetic link, so it is very unlikely the baby will also have it." 3. "It is rarely seen in boys, so it is not likely your new baby will have Hirschsprung disease." 4. "Hirschsprung disease is seen only in girls, so your new baby will not be at risk."

13. 1. CORRECT There is a genetic component to Hirschsprung disease, so any future siblings are also at risk. 2. There is a genetic component to Hirschsprung disease. 3. Hirschsprung disease is seen more commonly in males than females. 4. Hirschsprung disease is seen in both males and females but is more common in males.

14. The nurse is caring for an infant newly diagnosed with Hirschsprung disease. What does the nurse understand about this infant's condition? 1. There is a lack of peristalsis in the large intestine and an accumulation of bowel contents, leading to abdominal distention. 2. There is excessive peristalsis throughout the intestine, resulting in abdominal distention. 3. There is a small-bowel obstruction leading to ribbon-like stools. 4. There is inflammation throughout the large intestine, leading to accumulation of intestinal contents and abdominal distention.

14. 1. CORRECT In Hirschsprung disease, a portion of the large intestine has an area lacking in ganglion cells. This results in a lack of peristalsis as well as an accumulation of bowel contents and abdominal distention. 2. There is a lack of peristalsis at the aganglionic section of the bowel. 3. Hirschsprung disease does not include a small-bowel obstruction. 4. Hirschsprung disease does not present

14. The parents of a child hospitalized with minimal change nephrotic syndrome (MCNS) ask why the last blood test revealed elevated lipids. Which is the nurse's best response? 1. "If your child had just eaten a fatty meal, the lipids may have been falsely elevated." 2. "It's not unusual to see elevated lipids in children because of the dietary habits of today." 3. "Since your child is losing so much protein, the liver is stimulated and makes more lipids." 4. "Your child's blood is very concentrated because of the edema, so the lipids are falsely elevated."

14. 1. In MCNS, the lipids are truly elevated. Lipoprotein production is increased because of the increased stimulation of the liver hypoalbuminemia. 2. The elevated lipids are unrelated to the child's dietary habits. 3. CORRECT In MCNS, the lipids are truly elevated. Lipoprotein production is increased because of the increased stimulation of the liver hypoalbuminemia. 4. The lipids are not falsely elevated.

16. A parent asks the nurse how to prevent the child from having minimal change nephrotic syndrome (MCNS) again. Which is the nurse's best response? 1. "It is very rare for a child to have a relapse after having fully recovered." 2. "Unfortunately, many children have cycles of relapses, and there is very little that can be done to prevent it." 3. "Your child is much less likely to get sick again if sodium is decreased in the diet." 4. "Try to keep your child away from sick children because relapses have been associated with infectious illnesses."

16. 1. It is not unusual for a child to experience relapses. 2. Many children do experience relapses, but exposure to infectious illnesses has been linked to relapses. 3. There is no correlation between the consumption of sodium and nephrotic syndrome. 4. CORRECT Exposure to infectious illness has been linked to the relapse of nephrotic syndrome.

11. Which of the following is a (are) reason(s) to do a lumbar puncture on a child with a diagnosis of leukemia? Select all that apply. 1. Rule out meningitis. 2. Assess the central nervous system for infiltration. 3. Give intrathecal chemotherapy. 4. Determine increased intracranial pressure. 5. Stage the leukemia.

2. A lumbar puncture is done to assess the central nervous system by obtaining a specimen that can determine the presence of leukemic cells. 3. Chemotherapy can also be given with a spinal tap.

27. Which should be the priority nursing diagnosis for a 12-hour-old newborn with a myelomeningocele at L2? 1. Altered bowel elimination related to neurological deficits. 2. Potential for infection related to the physical defect. 3. Altered nutrition related to neurological deficit. 4. Disturbance in self-concept related to physical disability.

2. Potential for infection related to the physical defect.

20. A preschooler has been having periods during which he suddenly falls and appears to be weak for a short time after the event. The preschool teacher asks what she should do. Select the nurse's best response. 1. "Have the parents follow up with his pediatrician as this is likely an atonic seizure." 2. "Find out if there have been any new stressors in his life, as it could be attention-seeking behavior." 3. "Have the parents follow up with his pediatrician as this is likely an absence seizure." 4. "The preschool years are a time of rapid growth, and many children appear clumsy. It would be best to watch him, and see if it continues."

20. 1. CORRECT An atonic seizure is characterized by a loss of muscular tone, whereby the child may fall to the ground. 2. It is important to evaluate the child for life stressors, but suspected seizure activity needs immediate evaluation. 3. An absence seizure is characterized by a change in activity whereby the child appears to be daydreaming or staring straight ahead. The child usually continues basic simple movements but loses an awareness of surroundings. 4. The preschool years are a not a time of rapid growth. Many children in this age group appear clumsy, but suspected seizure activity needs immediate evaluation.

23. A child with hemolytic uremic syndrome (HUS) is very pale and lethargic. Stools have progressed from watery to bloody diarrhea. Blood work indicates low hemoglobin and hematocrit levels. The child has not had any urine output in 24 hours. The nurse expects administration of blood products and what else to be added to the plan of care? 1. Initiation of dialysis. 2. Close observation of the child's hemodynamic status. 3. Diuretic therapy to force urinary output. 4. Monitoring of urinary output.

23. 1. CORRECT Because the child is symptomatic, dialysis is the treatment of choice. 2. Because the child is symptomatic, dialysis is the treatment of choice. 3. Diuretics are given to prevent fluid overload, but they cannot cause the child in renal failure to produce urine. 4. Clotting factors are not used in HUS. The nurse would expect the plan to include dialysis, because the child is no longer producing urine.

23. The nurse is providing discharge teaching to the parents of a toddler who experienced a febrile seizure. The nurse knows clarification is needed when the mother says: 1. "My child will likely have another seizure." 2. "My child's 7-year-old brother is also at high risk for a febrile seizure." 3. "I'll give my child acetaminophen when ill to prevent the fever from rising too high too rapidly." 4. "Most children with febrile seizures do not require seizure medicine."

23. 1. Children who experience a febrile seizure are likely to experience another febrile seizure. 2. CORRECT Most children over the age of 5 years do not have febrile seizures. 3. Antipyretics are administered to prevent the child's temperature from rising too rapidly. 4. Most children are not prescribed anticonvulsant medication after experiencing a febrile seizure.

24. A child recently diagnosed with epilepsy is being evaluated for anticonvulsant medication therapy. The child will likely be placed on which type of regimen? 1. Two to three oral anticonvulsant medications so that dosing can be low and side effects minimized. 2. One oral anticonvulsant medication to observe effectiveness and minimize side effects. 3. One rectal gel to be administered in the event of a seizure. 4. A combination of oral and intravenous anticonvulsant medications to ensure compliance.

24. 1. Although many children with epilepsy require more than one medication to achieve seizure control, it is recommended that only one medication be started at a time so that the child's reaction to the specific medication can be observed. 2. CORRECT One medication is the preferred way to achieve seizure control. The child is monitored for side effects and drug levels. 3. Rectal gels are used to stop a seizure once it has begun; they are not used to prevent seizures. 4. The route of choice for the prevention of seizures is oral. There is no reason to assume that compliance will be an issue prior to beginning anticonvulsant therapy.

26. Which is the best action for the nurse to take during a child's seizure? 1. Administer the child's rescue dose of oral Valium (diazepam). 2. Loosen the child's clothing, and call for help. 3. Place a tongue blade in the child's mouth to prevent aspiration. 4. Carry the child to the infirmary to call 911 and start an intravenous line.

26. 1. Nothing should be placed in the child's mouth as he is at risk for aspiration. Rescue Valium is usually administered rectally. 2. CORRECT The nurse should remain with the child and observe the seizure. The child should be protected from his environment, and clothing should be loosened. 3. A tongue blade should never be placed in the child's mouth, as it can cause injury or increase the risk of aspiration. 4. The nurse should remain with the child and call for help. A child can be injured if carried during a seizure.

28. A child fell off his bike and sustained a closed-head injury. The child is currently awake and alert, but his mother states that he "passed out" for approximately 2 minutes. The mother appears highly anxious and is very tearful. The child was not wearing a helmet. Which is a priority for the triage nurse to say at this time? 1. "Was anyone else injured in the accident?" 2. "Tell me more about the accident." 3. "Did he vomit, have a seizure, or display any other behavior that was unusual when he woke up?" 4. "Why was he not wearing a helmet?"

28. 1. It is not a priority of care to find out if anyone else was injured. 2. Although open-ended questions are important, the nurse needs specific information, and the anxious parent may need to be guided during triage assessment. 3. CORRECT Asking specific questions will give the nurse the information needed to determine the level of care for the child. 4. Although it is important to provide safety education, this information should be given in a nonjudgmental manner at a point when the parents and child are less stressed.

The finding the nurse would expect when measuring blood pressure on all four extremities of a child with coarctation of the aorta is blood pressure that is

Lower in the legs than in the arms.

48. The nurse is caring for a 2-month-old infant who is at risk for cerebral palsy (CP) due to extreme low birth weight and prematurity. His parents ask why a speech therapist is involved in his care. Select the nurse's best response. 1. "Your child is likely to have speech problems because of his early birth. Involving the speech therapist now will ensure vocalization at a developmentally appropriate age." 2. "The speech therapist will help with tongue and jaw movements to assist with babbling." 3. "The speech therapist will help with tongue and jaw movements to assist with feeding." 4. "Many members of the health-care team are involved in your child's care so that we will know if there are any unmet needs."

3. "The speech therapist will help with tongue and jaw movements to assist with feeding."

39. The parent of an infant asks the nurse what to watch for to determine if the infant has CP. Which is the nurse's best response? 1. "If the infant cannot sit up without support before 8 months." 2. "If the infant demonstrates tongue thrust before 4 months." 3. "If the infant has poor head control after 2 months." 4. "If the infant has clenched fists after 3 months."

4. "If the infant has clenched fists after 3 months."

85. Parents of a child with generalized seizures ask the nurse for information to give their child's teachers. Which of the following should be included? 1. A soft-padded spoon should be kept nearby to put between the child's teeth at the onset of a seizure. 2. Roll the child onto the abdomen, with the head to the left, so any contents can flow from the mouth. 3. If a seizure lasts longer than 10 minutes, the parents or an ambulance should be called. 4. As the child grows, medication dosages may need to be adjusted to control seizure activity

4. As the child grows, medication dosages may need to be adjusted to control seizure activity

61. Which child may need extra fluids to prevent dehydration? Select all that apply. 1. 7-day-old receiving phototherapy. 2. 6-month-old with newly diagnosed pyloric stenosis. 3. 2-year-old with pneumonia. 4. 2-year-old with full-thickness burns to the chest, back, and abdomen. 5. 13-year-old who has just started her menses.

61. 1, 2, 3, 4. 1. The lights in phototherapy increase insensible fluid loss, requiring the nurse to monitor fluid status closely. 2. The infant with pyloric stenosis is likely to be dehydrated due to persistent vomiting. 3. A 2-year-old with pneumonia may have increased insensible fluid loss due to tachypnea associated with respiratory illness. The nurse needs to monitor fluid status cautiously because fluid overload can result in increased respiratory distress. 4. The child with a burn experiences extensive extracellular fluid loss and is at great risk for dehydration. The younger child is at greater risk due to greater proportionate body surface area. 5. An adolescent starting her menses is not at risk for dehydration.

A pediatric nurse is caring for a patient who received chemotherapy 10 days ago. Which laboratory value requires the nurse's intervention? A blood urea nitrogen level of 10 mg/dL A hemoglobin of 8.6 g/dL A platelet count of 18 x 103/µL A serum glucose of 110 mg/dL

A platelet count of 18 x 103/µL

5. An infant is experiencing dyspnea related to patent ductus arteriosus (PDA). The nurse understands dyspnea occurs because blood is: a. circulated through the lungs again, causing pulmonary circulatory congestion. b. shunted past the pulmonary circulation, causing pulmonary hypoxia. c. shunted past cardiac arteries, causing myocardial hypoxia. d. circulated through the ductus from the pulmonary artery to the aorta, bypassing the left side of the heart.

ANS: A When PDA is present, oxygenated blood recycles through the lungs, overburdening the pulmonary circulation.

The nurse is caring for an infant whose cleft lip was repaired. Important aspects of this infant's postoperative care include which of the following? a. Arm restraints, postural drainage, mouth irrigations b. Cleansing suture line, supine and side-lying position, appropriate analgesia c. Mouth irrigations, prone position, cleansing suture line d. Supine and side-lying positions, postural drainage, arm restraints

ANS: B The suture line should be cleansed gently after feeding. The child should be positioned on back, on side, or in infant seat. The child is medicated with appropriate analgesia to calm him or her. Postural drainage is not indicated. This would increase the pressure on the operative site when the child is placed in different positions. There is no reason to perform mouth irrigations, and the child should not be placed in the prone position where injury to the suture site can occur. Arm restraints are used according to local practice.

3. The finding the nurse would expect when measuring blood pressure on all four extremities of a child with coarctation of the aorta is blood pressure that is: a. higher on the right side. b. higher on the left side. c. lower in the arms than in the legs. d. lower in the legs than in the arms.

ANS: D The characteristic symptoms of coarctation of the aorta are a marked difference in blood pressure and pulses between the upper and lower extremities. Pressure is increased proximal to the defect and decreased distal to the coarctation.

The nurse is providing education to the parents of a teenaged boy diagnosed with impetigo. Which of the following statements by the boy indicates the need for further education? a) "This condition is contagious." b) "I will need to cover my son's skin lesions with bandages until it has healed." c) "It is important to remove the crusts before applying any topical medications." d) "My son can continue to attend school while he is taking the prescribed antibiotics."

b)"I will need to cover my son's skin lesions with bandages until it has healed." Explanation: Impetigo is an infectious bacterial infection. The crusts should be removed after soaking prior to applying topical medications. Leaving the lesions open to air is not contraindicated. Children diagnosed with impetigo may attend school during treatment.

The nurse is caring for a child admitted with partial thickness burns. Which of the following is most characteristic of this type of burn? a) Skin is red and edematous b) Muscle damage occurs c) Blisters appear d) Pain is minimal

c)Blisters appear Explanation: In partial-thickness or second-degree burns, blistering usually occurs with an escape of body plasma.

The nurse is caring for a child with a partial-thickness burn. Which of the following assessment findings would the nurse expect to observe? a) Peeling skin with eschar b) Edema with dry or waxy-looking skin c) Edema with wet blistering skin d) Reddened and leathery skin

c)Edema with wet blistering skin Explanation: Partial-thickness burns are very painful and edematous and have a wet appearance or the presence of blisters. Full-thickness burns appear red, edematous, leathery, dry, or waxy and may display red or charred skin (eschar).

The nurse is caring for a 7-year-old with burns. Which finding would be highly suggestive of a child abuse-induced burn? a) Spattering pattern b) Splash patterns c) Stocking-glove pattern on hands or feet d) Nonuniform pattern

c)Stocking-glove pattern on hands or feet Explanation: A stocking-glove pattern on the hands or feet or a circumferential ring appearing around the extremity points to the caregiver forcefully holding the child under extremely hot water. A nonuniform pattern and splash or spattering patterns are not typical indicators of child abuse.


Kaugnay na mga set ng pag-aaral

Georgia law, rules and regulations

View Set

Test 4: Chapters 6,7,8 (Macro Economics)

View Set

Act cH4 + subject verb agreement

View Set

Care & Prevention of Athletic Injury Quiz 2

View Set

Genetics Chapter 1: Overview of Genetics

View Set